¿Es compatible el tercero de Newton con la fuerza de Lorentz retardada?

En la Introducción a la electrodinámica de Griffiths se dice que la tercera ley de Newton no es válida en electrodinámica, pero, en el ejemplo dado, no considera los valores retardados de los campos y observa la contradicción solo en la fuerza magnética.

Simplemente poniendo el ejemplo de los libros: dos cargas positivas se mueven a lo largo de los ejes x e y hacia el origen. Las fuerzas eléctricas cumplen la tercera ley de Newton, pero las magnéticas no.

A continuación, una imagen con el ejemplo original y un 'diagrama' que muestra de dónde proviene mi duda:1

Pregunta: ¿Se cumple la tercera ley de Newton para la fuerza de Lorenz con campos retardados apropiados?

Respuestas (2)

Tus imágenes no explican muy bien cuál es tu duda.

Si las cargas se movieron uniformemente durante el tiempo suficiente en el pasado, las fuerzas eléctricas están a lo largo de la línea de unión de las partículas, incluso si se tiene en cuenta el retardo. Los campos magnéticos son perpendiculares a las velocidades, por lo que existen arreglos en los que el momento de las partículas no se conserva; tal vez haya alguna especial en la que se conserve el impulso. ¿Puedes encontrar una?

Si las partículas aceleran, ni siquiera los campos eléctricos están a lo largo de la línea de unión, y las fuerzas pueden no ser ni siquiera de la misma magnitud. En tal caso, en general, el momento de las partículas no se conservará.

Si el momento electromagnético se define en base a las ecuaciones de Maxwell y la fórmula de la fuerza de Lorentz, y si el momento total lo incluye, el momento total aún se conserva, pero el momento electromagnético no será tan "visible", ya que no es un momento ordinario de ningún tipo. partícula, pero se distribuye en todo el espacio, la mayor parte cerca de las partículas.

"Si las cargas se movieron uniformemente durante el tiempo suficiente en el pasado, las fuerzas eléctricas están a lo largo de la línea de unión de las partículas, incluso si se tiene en cuenta el retardo" << Esta oración por sí sola resuelve la duda. Creo que por eso te pareció extraño mi dibujo, porque está mal en ese punto.

Para la primera figura que se muestra, y suponiendo magnitudes de velocidad iguales para ambas cargas, obtengo que la tercera ley de Newton no se cumple porque:

1) La fuerza eléctrica de la carga 2 sobre la carga 1 es igual en magnitud y de dirección opuesta a la de la carga 1 sobre la carga. Ambos tienen componentes x e y.

2) La fuerza magnética de la carga 2 sobre la carga 1 es igual en magnitud a la de la carga 1 sobre la carga 2. La primera tiene solo componente y y la última solo componente x.

3) La fuerza total de la carga 2 sobre la carga 1 es igual en magnitud a la de la carga 1 sobre la carga 2. Sin embargo, la primera y la última solo estarán en direcciones aproximadamente opuestas para velocidades de carga pequeñas en comparación con c.

4) Tenga en cuenta que, en un punto de campo dado (x,t) o (y,t), los campos eléctrico y magnético no dependen de los atributos de la carga que pasa por ese punto, es decir, su velocidad o ecuación de movimiento. .

OP parece estar interesado en E&M no relativista, por lo que no estoy seguro de por qué lo presenta en (3).